Please help i need to find X.

Please Help I Need To Find X.

Answers

Answer 1

Answer:

(11√42)/2

Step-by-step explanation:

Thanks to the given information we can say that the smaller leg of the left triangle is

(11√7)/2

while the other leg is

11/2 √7 * √3 = 11/2 √21

the triangle on the right has two congruent legs. For the Pythagorean theorem

x^2 = 2 (11/2 √21)^2

x^2 = 2  * 121/4 * 21

x^2 = 2541 / 2

x = √11^2 * 3 * 7/√2

x = 11√21/√2

x = (11√42)/2


Related Questions

William needs to work out the size of angle Y in this diagram

One of William’s reasons are wrong.

Write down the correct reason.

Answers

Answer:

because internal staggal angles are equal

Step-by-step explanation:

The first reason is wrong.

Angle EGH and DEG are internal staggal angles:

the two angles are on both sides of the cut line EG, and the two angles are between the two divided lines.

{the definition of internal staggal angle}

Which of the following is the explicit rule for a geometric sequence defined by
a recursive formula of a, - 138-1 for which the first term is 7?

Answers

Answer:

C

Step-by-step explanation:

What you wrote is not the same thing as what the question is, or at least I don't think so. I'll answer the printed question.

First of all, the 13 is what separates each of the terms. In other words 7 is the first term. 13 must be raised to the n - 1 power.

It is written like this

an = 7 * 13^(n - 1)

you want a1 to be 7. The only way that can happen is if 13^0 which gives you 1.

So the correct answer is C

Determine the length of AC.

32 units
35.2 units
38.5 units
10.3 units

Answers

Answer:

38.5

Step-by-step explanation:

b^2= 29^2+20^2-2×20×29× Cos 102

b^2=841+400-1160 (-0.2079)

=1241- 241.164

b^2=1482.164

b= square root of 1482.164

AC = 38.5

What is the value of x? Show work.

Answers

Answer:

x=5

Step-by-step explanation:

This is because the inscribed angle of the circle or angle EFG is equal to the twice of the degree of the arc EG. Therefore, we can create this equation:

12+40=2(8x+10)

12+40=16x+20

20=4x

x=5

An operator in the plant receives a monthly salary. His rent, which is $849, takes up exactly 1/3 13 of his pay. What is his total pay per month?

Answers

Answer:

Below.

Step-by-step explanation:

(1/3)X = 849

x = 849*3 = 2547

The total pay per month of an operator is equal to $2547.

What is an expression?

The mathematical expression combines numerical variables and operations denoted by addition, subtraction, multiplication, and division signs.

Mathematical symbols can be used to represent numbers (constants), variables, operations, functions, brackets, punctuation, and grouping. They can also denote the logical syntax's operation order and other properties.

Given that an operator in the plant receives a monthly salary. His rent, which is $849, takes up exactly 1/3.

The total monthly salary will be calculated as,

(1/3)X = 849

X = 849 x 3

X= 2547

Therefore, the total pay per month of an operator is equal to $2547.

To know more about an expression follow

https://brainly.com/question/20912045

#SPJ5

Evaluate the expression.
32 + 6 x 22-42 - 23

Answers

Answer:

25

Step-by-step explanation:

You need to simplify

.

.

.

.

................... :)

Answer:

D

Step-by-step explanation:

9+24-16+8= 25

If €1 is £0.72
What is £1 in €

Answers

Answer:

€1.39

Step-by-step explanation:

£0.72 ÷ 0.72 = 1÷0.72

£1= €1.39

How do I do this problem?

Answers

Step-by-step explanation:

here is the answer to your question

Answer:

Step-by-step explanation:

I would start from the beginning and find the slope myself, just so I know what's going on (as opposed to being dropped in the middle of the problem). The slope formula is:

[tex]m=\frac{y_2-y_1}{x_2-x_1}[/tex] and for us:

[tex]m=\frac{5-3}{4-3}=2[/tex] so the slope is indeed 2. Now we need to write the equation in slope-intercept form. I find it easier to first write the equation in point-slope form and then solve it for y. Point-slope form is

[tex]y-y_1=m(x-x_1)[/tex] where m is the slope (2) and x1 and y 1 are from one of the coordinates (whichever one you want; as long as you do the math correctly, you will NOT get an incorrect answer. In other words, you can't pick the "wrong" point to use to write the equation.) I'm going to use (3, 3):

y - 3 = 2(x - 3) and

y - 3 = 2x - 6 and

y = 2x - 6 + 3 so

y = 2x - 3 and that's your equation. Of course, you will enter a (-3) in that box with the ? in it.

Please help me....Which inequality is true?

Answers

Answer:

opo

Step-by-step explanation:

kung Hindi kayo sure sa sagot ko sorry po

Find the least number which should be added to 6790 to make it a perfect square​

Answers

Answer:

add 99 to 6790

Step-by-step explanation:

6790 +99 = 6889 which is 83 squared

Find the length of side AB.
Give your answer to 1 decimal place.
12 cm
62°
A
B

Answers

Hey there!

To solve this problem, we will be using Trigonometric Ratio. Trigonometry is always helpful when it comes to finding a missing side with specific measure/angle.

1. Cosine Ratio

Currently, there are 6 Trigonometric Ratios. But we will be talking about Cosine Ratio instead since it is what we will be using in your question! Cosine Ratio is defined as adjacent to hypotenuse or adjacent/hypotenuse. You know what adjacent and hypotenuse are right? If not then head to the next topic!

2. Adjacent and Hypotenuse

Adjacent is basically the base of a triangle. It is basically drawn from right angle to any measure/angles. Hypotenuse is the longest side of a right triangle. It is also an opposite side of right angle.

Hope you understand this topic! If not, feel free to ask!

3. Solve The Problem

Since our adjacent is length AB but we don't know its exact value. What we have to do is to determine AB as any variables which I will determine AB as "x". Next, we have the value of hypotenuse which is 12 cm. Then we also know the Cosine Ratio which is adjacent to hypotenuse.

Therefore, the equation for the problem is:

[tex] \large{cos62 \degree = \frac{x}{12} }[/tex]

*cos is the short form of cosine*

At this part, we need a calculator to find the value of cos62 degrees. That's because it is not a degree like 0, 30, 45, 60 and 90 which can be found without a calculator.

When we put cos62 in a calculator, make sure to put it in degree mode since a calculator has two modes which are degree and radian.

When we put cos62 in, we should get 0.46947156... Because you want a one decimal place, we round the value up to the nearest tenth as we get 0.5 because 6 is greater than 5 and should be rounded up and not down. That makes the equation to:

[tex] \large{0.5 = \frac{x}{12} }[/tex]

Oh well! Finally to the equation part. Whenever you have to solve the equation that has decimal numbers in it, the best way to deal with decimal numbers is to make them into a whole number or integer. But how? Simply multiply the whole equation by 10. Because 0.5×10 is 5 thus 0.5 becomes an integer after multiplying 10.

[tex] \large{0.5 \times 10 = \frac{x}{12} \times 10} \\ \large{5 = \frac{10x}{12} }[/tex]

10x/12 can be simplified again.

[tex] \large{5 = \frac{5x}{6} }[/tex]

Then we isolate x-value by multiplying 6 the whole equation.

[tex] \large{5 \times 6 = \frac{5x}{6} \times 6} \\ \large{30 = 5x} \\ \large{x = 6}[/tex]

Huh, that's awkward! We want the answer in a 1 decimal place but seems like the answer for this is 6. Why? Well that is not an exact answer, but more like an approximation. Because the value of cos62 degree is actually a repeating decimal and doesn't have exact value.

When we put the equation cos62 = x/12 in the equation and solve. It appears that the the answer is 5.63365875. Because we round up to nearest tenth, it gives an approximation to 5.63365875 instead.

Hence, the equation and value above is just a rounded to the whole number from 5.63365875.

Because you want a one place decimal. Hence,

4. Final Answer

The length AB is 5.6 (rounded to nearest tenth)

the base of a right prism is an equilateral triangle each of whose sides measures 4cm.the altitude of the prism measures 5cm.Find the volume of the prism ​

Answers

Answer:

[tex]V=34.64\ cm^3[/tex]

Step-by-step explanation:

Given that,

The side of an equilateral prism = 4 cm

The altitude of the prism = 5 cm

We need to find the volume of the prism. The formula for the volume of a prism is as follows :

[tex]V=A\times h[/tex]

Where

A is the area of equilateral triangle, [tex]A=\dfrac{\sqrt3}{4}a^2[/tex]

So,

[tex]V=\dfrac{\sqrt3}{4}a^2\times h\\\\V=\dfrac{\sqrt3}{4}\times 4^2\times 5\\\\V=34.64\ cm^3[/tex]

So, the volume of the prism is equal to [tex]34.64\ cm^3[/tex].

c
Question 15
1 pts
What is the sum of (3x² – 4x + 8) and
(-22 – 2x – 8) ?
2.02 - 60 + 1
o
222 - 2x
O 2x2
6x
O 2x2 + 62

Answers

Answer:

2x² - 6x

Step-by-step explanation:

(3x² - 4x + 8) + (- x² - 2x - 8) ← remove both parenthesis

= 3x² - 4x + 8 - x² - 2x - 8 ← collect like terms

= 2x² - 6x

Which explains whether or not her solutions are correct? (Algebra ll) *URGENT*

Answers

Answer:

I believe its A

Step-by-step explanation:

Substitue vales into the equation and check! Lmk if it was ccorrect!

What is the solution to the equation below. Round your answer to two
decimal places
In x= 3.1

Answers

The solution to the equation below rounded to 2 decimal places is 22.20.

How to simplify logarithmic equations?

Given the logarithmic expressions

ln x = 3.1

We are to determine the value of "x"

ln x = 3.1

Take the exponent of both sides

e^ln x = e^3.1

The exponent will cancel out the log function to have:

x = e^3.1

x = 22.197

Hence the solution to the equation below rounded to 2 decimal places is 22.20.

Learn more on log function here: https://brainly.com/question/1695836

s part of a science experiment, Natasha drops a bouncy ball from various heights, h, and observes the height to which the ball rebounds, r. The table shows the results of Natasha's experiment.

Initial height, h (in meters) Rebound height, r (in meters)
0.30 0.24
0.50 0.4
0.80 0.64
1.00 0.8
1.20 0.96

Answers

Answer: r = 0.6h

your welcome :D

Answer:

The actual correct answer is r = 0.8h

Step-by-step explanation:

You divide 0.24 ÷ 0.30 and get 0.8

Do the same thing for all the rest of the numbers but,

you must put the numbers that are on the right FIRST then divide them by the numbers on the left, all of these are the same answer which means it's

r = 0.8h

Hope this helps!

plz do this i will mark you

Answers

Photo isn't clear.

And which number

write the following expressions as an exponential term with a single base
(4*2^5)÷(2^3 *1/16)

Answers

Answer:

2^8

Step-by-step explanation:

(4*2^5)÷(2^3 *1/16)

[tex]\left ( 4\times 2^5 \right )\div \left ( 2^3\times \frac{1}{16} \right )\\\\2^7 \div 2^{-1}\\\\2^7\times 2^1\\\\2^8[/tex]

Help and explain pls and ty

Answers

Answer:

(g ○ f)(3) = 23

Step-by-step explanation:

Evaluate f(3) then substitute the value obtained into g(x)

f(3) = 2(3) + 4 = 6 + 4 = 10 , then

g(10) = 3(10) - 7 = 30 - 7 = 23

The 9th term of an arithmetic progression is 3+3p and the sum of the first four terms is 2p-10, where p is constant. Given that the common difference is 2, find the value of p.​

Answers

Answer:

3

Step-by-step explanation:

Let's find the first term in terms of p.

So an arithmetic sequence is a linear relation.

That means it will have the same slope no matter the pair of points used. We are given the slope, the common difference, is 2. We are going to use point (9, 3+3p) and (1, t(1)) along with m=2 to find t(1).

[t(1)-(3+3p)]/[1-9]=2

Simplify denominator

[t(1)-(3+3p)]/[-8]=2

Multiply both sides by -8

t(1)-(3+3p)=-16

Add (3+3p) on both sides

t(1)=-16+3+3p

Combine like terms

t(1)=-13+3p

This means we can find the next term by adding 2 this.

t(2)=-11+3p

Let's find the next term by adding 2 this.

t(3)=-9+3p

Finally we can find the 4th term by adding 2 to this

t(4)=-7+3p

We are given the sum of the first 4 terms is 2p-10. So we can write:

-13+3p+-11+3p+-9+3p+-7+3p=2p-10

Combine like terms on left

12p-40=2p-10

Subtract 2p on both sides

10p-40=-10

Add 40 on both sides

10p=30

Divide both sides by 10

p=3.

-----------------

Checking:

t(1)=-13+3p=-13+3(3)=-13+9=-4

t(2)=-11+3p=-11+3(3)=-11+9=-2

t(3)=-9+3p=-9+3(3)=-9+9=0

t(4)=-7+3p=-7+3(3)=-7+9=2

----sum of the first 4 is -4

And 2p-10 at p=3 gives 2(3)-10=6-10=-4

So this part checks out

In general, the pattern that those 4 terms I wrote out follow t(n)=(-13-2)+2n+3p. I know this because the 0th term would have been (-13-2)+3p and this part goes up by 2 each time. The plus 3p part doesn't change.

Anyways t(9)=(-13-2)+2(9)+3p=-15+18+3p=3+3p. And this part looks good too.

2m^2-5m-3=0 by factorization​

Answers

Step-by-step explanation:

It is so simple Hope u understand

Answer:

Step-by-step explanation:

Sum = -5

Product = 2*(-3) = -6

Factors = -6 , 1   {-6 + 1 = -5  & -6 *1 = -6}

2m² - 5m -3 = 0

2m² - 6m + m -3 = 0

2m(m - 3) + (m -3) = 0

(m -3)(2m + 1) = 0

m - 3 =  0    or 2m + 1 = 0

    m = 3      or 2m = -1

                          m = -1/2

Ans: m = 3 , (-1/2)

!!!PLEASE HELP!!!
explain the error

Answers

Answer:

Step-by-step explanation:

x is greater than minus 3. You have everything right but the graph. The arrowhead goes the other way. Around -3 is as small as x gets.

0====0====0====0====0

-3       -2      -1        O       1

O------------------------------->

find perimeter of the semicircular region​

Answers

Answer:

4. 87.96 yd

5. 47.12 ft

6. 34.56 cm

Step-by-step explanation:

4. diameter(d) = 28 yd,

perimeter = πd = 87.96 yd (rounded to 2 dp)

5. radius(r) = 7.5 ft

perimeter = 2πr = 47.12 ft (rounded to 2 dp)

6. diameter (d) = 11 cm

perimeter = πd = 34.56 cm

Answered by GAUTHMATH

Formula to find the perimeter of a semicircular region:

if radius given: [tex]P=2r[/tex] +  [tex]\pi r[/tex]  {r = radius}

if diameter given: [tex]P=d+\pi r[/tex] {d = diameter}

Take [tex]\pi[/tex] as 3.14

Q4.

Diameter = 28 yd

Radius = 28/2 = 14 yd

[tex]P=28+\pi (14)[/tex]

[tex]P=28+(3.14)(14)[/tex]

[tex]P=28+14.96[/tex]

[tex]P=42.96[/tex] yd

Q5.

Radius = 7.5 ft

Diameter = 7.5 × 2 = 15 ft

[tex]P=2(7.5)+\pi (7.5)[/tex]

[tex]P=2(7.5)+(3.14)(7.5)[/tex]

[tex]P=15+23.55[/tex]

[tex]P=38.55[/tex] ft

Q6.

Diameter = 11 cm

Radius = 11/2 = 5.5 cm

[tex]P=11+\pi (5.5)[/tex]

[tex]P=11+(3.14)(5.5)[/tex]

[tex]P=11+17.27[/tex]

[tex]P=17.27[/tex] cm

I hope this helps...

Have a great day ahead :)

Division method of 298116 using short Division method​

Answers

Answer:

I think you are being asked to evaluate the square root of 298116 by division method. Here is how its done:

546

51298116

125

104 81 10416

10861 0 0 6516 1006516

Please feel free to ask if something is not clear.

Here are the steps you will need to follow:

Step 1:

We find the largest square smaller than 29, that's 25. Subtract the 25 from the 29, which leaves 4, and enter a 5 as the first digit of your quotient.

Step 2:

104

Find the area of the rhombus

Answers

Answer:

The rhombus has an area of 240 units. (2nd one on the left).

Step-by-step explanation:

The intersection of diagonals meet at right angles. That means that the labeled sides are the hypotenuse and 1 leg of a right angle triangle. You need to find the other leg by using the Pythagorean Theorem

a^2 + b^2 = c^2

a = 8

b = ?

c = 17

8^2 + b^2 = 17^2

64 + b^2 = 289                 Subtract 64

b^2 = 289-64

b^2 = 225                         Take the square root of both sides.

sqrt(b^) = sqrt(225)

b = 15

Find the area of 1 triangle

Area = 1/2 * a * b

a = 8

b = 15

Area = 1/2 * 8 * 15

Area = 60

Now find the area of 4 triangles.

4*60 = 240

280L of water consumed my 7 people. water consumed by 50 people =___L

Answers

Step-by-step explanation:

7 people = 280 liters

1 p = 40 liters

50 p = 40 x 50

50 p = 2000 liters

hope it helps.

Also, I think that Brainly is an awesome app, but there's an app which is doing great work for me in maths, named Gauthmath. I will suggest it. Video concepts and answers from real tutors.

Find all solutions to the equation.
cos x = sin x (6 points)

Answers

Answer:

Step-by-step explanation:

cos x=sinx

divide by cos x

1=tan x

tan x=1

[tex]tan x=1=tan \frac{\pi }{4}=tan (n\pi +\frac{\pi }{4})\\x=n\pi +\frac{\pi }{4} ,\\where~n~is~an~integer.[/tex]

Thank you very much you can help me with the others that I am uploading to my profile and there is more in case you can please

Answers

Answer: y = 2x + 20; x is any real number greater than or equal to 0, and y is any real number greater than or equal to 20.

Step-by-step explanation:

Because it's time, it don't have to be just integers. Seconds are fractions of minutes.(0,20) is the y-intercept and there will be no smaller y-values because x(time) cannot go negative.He goes at a rate of a problem per 2 minutes, so 2 is his slope.

Answer:

The second one is the answer.

Step-by-step explanation:

Objective: Linear Equation Problems.

Let model the equation given to us.

Vlad spent 20 minutes on his homework already and he complete x math problems every 2 minutes so we represent this as

[tex]y = 2x + 20[/tex]

X can be any real number greater than or equal zero because time isnt negative so anything below zero is wrong.

Y can be any real number but however since Vlad spent 20 mins on his homework already, he must spend over or equal to 20 minutes completing the total homework.

The second one is the answer

The difference between two numbers is 12 and their sum is 26. Find the smallest number. A)9 B)7 C)-7 D)-9

Answers

Answer:

Step-by-step explanation:

We need a system of equations to solve for this. The first equation comes from "The difference between 2 numbers is 12". Difference means subtraction, the 2 numbers will be x and y, and the word "is" means equals:

x - y = 12

The second equation comes from "...their sum is 26". The numbers are still x and y but this time we have their sum, which is what they equal when they are added together. The word "is" means equals:

x + y = 26. So we will go back up to the first equation. If y taken away from x leaves a positive number, that means that y is smaller than x. We are looking for the value of the smaller number so we will find y. Solve the first equation for x to get it in terms of y:

x = 12 + y and we sub that into the second equation:

(12 + y) + y = 26 and

12 + 2y = 26 and

2y = 14 so

y = 7. Choice B is the one you want.

work out the number of ham sandwiches Tina had

Answers

Answer:

Number of ham sandwiches Tina made = 60

Step-by-step explanation:

Total number of sandwiches = 480

Fraction of total sandwiches which are Tuna = [tex]\frac{3}{8}[/tex]

Therefore, number of Tuna sandwiches = [tex]480\times \frac{3}{8}[/tex]

                                                                   = 180

Percentage of beef sandwiches = 35%

Therefore, beef sandwiches = [tex]480\times \frac{35}{100}[/tex]

                                                = 168

Ratio of ham sandwiches (h) and cheese sandwiches (c) = 5 : 6

[tex]\frac{h}{c}=\frac{5}{6}[/tex]

[tex]c=\frac{6}{5}h[/tex] ------(1)

Total number of Tuna sandwiches + Total number of beef sandwiches + Total number of ham sandwiches + Total number of cheese sandwiches = 480

180 + 168 + h + c = 480

h + c = 480 - 348

h + c = 132

Substitute the value of c from equation (1),

[tex]h+\frac{6}{5}h=132[/tex]

[tex]\frac{11}{5}h=132[/tex]

[tex]h=\frac{132\times 5}{11}[/tex]

[tex]h=60[/tex]

Therefore, number of ham sandwiches Tina made = 60

Other Questions
Does this sound good for my character sketch?If you could, I'm open to suggestions. What are the nouns and verbs in the sentence."Ravens call each other with deep, rumbling croaks which vary in tone and length. what is the formula for the trainglar figures What is the vertex of the parabola?y - 3 = 1/2 (x + 5)( __ , __ ) Find the measure of the indicated side Don't squeeze. Fiver shoppers buy Charmain toilet paper. One charmin out of 10 in this batch is defective-its unsqueezable. You want to save every one from this catastrophe, so you stop them at the door and ask to squeeze their charmin. After squeezing 5 rolls, what is the possibility that you have located 1 or more defective charmins? Hint start off with the chance that you squeezed no defective rolls. Suppose the mean income of firms in the industry for a year is 95 million dollars with a standard deviation of 11 million dollars. If incomes for the industry are distributed normally, what is the probability that a randomly selected firm will earn less than 114 million dollars Why the mode of pollination in maize is cross-pollination? Why it is important to cover the ear shoot with a butter paper bag before the emergence of silks? What could be the consequences if a few silks emerge before bagging? (Marks4) If the limelight of a movie was on a character who had one defect, the scriptwriter might have been making an allusion to _____________. Gabe went out to lunch with his best friend. The bill cost $16.40 before tax and tip. He paid a 9% tax and he left a 20% tip. How much did Gabe spend? Hint: Tax and tip are both based on the original cost of the bill.Don't forget to round to the nearest cent! Which question cannot be answered through making measurements? A. Should the government invest in technologies that can reduce the rate of global warming? B. How much carbon dioxide is released into the atmosphere when a square mile of tropical rain forest burns? C. What is the relationship between carbon dioxide emissions and average global temperature? D. Which human activities contribute to global warming? 5. Rodney wants to Increase his physical activity to Improve his overall health. What change to his level of optimal wellness can Rodney expect as a result?A. Improved ability to relate to othersB. Immediate Improvements to IQ levelC. Improved mood/reduced level of anxietyD. reduced risk of lung cancer what is circle graph or pie chart? Glen Pool Club, Inc., has a $150,000 mortgage liabilty. The mortgage is payable in monthly installments of $1,543 , which include interest computed at an annual rate of 12 percent (1 percent monthly). Prepare a partial amortization table showing (1) the original balance of this loan, and (2) the allocation of the first two monthly payments between interest expense and the reduction in the mortgage`s unpaid balance. Prepare the journal entry to record the second monthly paymment. Will monthly interest increase, decrease or stay the same over the life of the loan? Explain. Which statement is true regarding the functions on thegraph?f(6) = g(3)f(3) = g(3)f(3) = g(6)f(6) = g(6) A robot that makes _/6 of a boat per day will make 5 boats in 6 days A species in imminent danger of extinction throughout all or a significant portion of its range is said to be: How is each triglyceride different from the others? if you are the principal of your school how do you manage all staffs I NEED HELP SOMEONE HELP ME OUTTT!!